Last visit was: 24 Apr 2024, 13:04 It is currently 24 Apr 2024, 13:04

Close
GMAT Club Daily Prep
Thank you for using the timer - this advanced tool can estimate your performance and suggest more practice questions. We have subscribed you to Daily Prep Questions via email.

Customized
for You

we will pick new questions that match your level based on your Timer History

Track
Your Progress

every week, we’ll send you an estimated GMAT score based on your performance

Practice
Pays

we will pick new questions that match your level based on your Timer History
Not interested in getting valuable practice questions and articles delivered to your email? No problem, unsubscribe here.
Close
Request Expert Reply
Confirm Cancel
SORT BY:
Kudos
Math Expert
Joined: 02 Sep 2009
Posts: 92902
Own Kudos [?]: 618802 [10]
Given Kudos: 81588
Send PM
GMAT Club Legend
GMAT Club Legend
Joined: 03 Jun 2019
Posts: 5343
Own Kudos [?]: 3964 [3]
Given Kudos: 160
Location: India
GMAT 1: 690 Q50 V34
WE:Engineering (Transportation)
Send PM
Senior Moderator - Masters Forum
Joined: 19 Jan 2020
Posts: 3137
Own Kudos [?]: 2769 [0]
Given Kudos: 1510
Location: India
GPA: 4
WE:Analyst (Internet and New Media)
Send PM
Intern
Intern
Joined: 28 Jan 2019
Posts: 24
Own Kudos [?]: 7 [0]
Given Kudos: 360
Location: India
Concentration: Leadership, Strategy
Send PM
Re: Is |x - 3| < 7 ? (1) x > 0 (2) x < 10 [#permalink]
yashikaaggarwal wrote:
Is |x - 3| < 7 ?

(1) x > 0
Case 1: put x= 1
|1-3|<7
2<7 (sufficient)

Case 2: put x = 12
|12-3|>7
9>7 (insufficient)

Case 3: put x=9
|9-3|=7
7=7(insufficient)

Statement 1: (insufficient alone)

(2) x < 10
Case 1: x=9
|9-3|=7
7=7 (insufficient)

Case 2: put x= 1
|1-3|<7
2<7 (sufficient)

Case 3: put x = -12
|-12-3|>7
15>7 (insufficient)
Statement 2 is insufficient alone.

Statement 1 and 2:
0<x<10
Case 1: x=9
|9-3|=7
7=7 (insufficient)

Case 2: put x= 1
|1-3|<7
2<7 (sufficient)

(Not sufficient)

Answer is E

Posted from my mobile device


Statement 1 and 2:
0<x<10
Case 1: x=9
|9-3|=7
7=7 (insufficient)

|9-3| is 6 - it's sufficient :) :)
Senior Moderator - Masters Forum
Joined: 19 Jan 2020
Posts: 3137
Own Kudos [?]: 2769 [0]
Given Kudos: 1510
Location: India
GPA: 4
WE:Analyst (Internet and New Media)
Send PM
Re: Is |x - 3| < 7 ? (1) x > 0 (2) x < 10 [#permalink]
Anuragjn wrote:
yashikaaggarwal wrote:
Is |x - 3| < 7 ?

(1) x > 0
Case 1: put x= 1
|1-3|<7
2<7 (sufficient)

Case 2: put x = 12
|12-3|>7
9>7 (insufficient)

Case 3: put x=9
|9-3|=7
7=7(insufficient)

Statement 1: (insufficient alone)

(2) x < 10
Case 1: x=9
|9-3|=7
7=7 (insufficient)

Case 2: put x= 1
|1-3|<7
2<7 (sufficient)

Case 3: put x = -12
|-12-3|>7
15>7 (insufficient)
Statement 2 is insufficient alone.

Statement 1 and 2:
0<x<10
Case 1: x=9
|9-3|=7
7=7 (insufficient)

Case 2: put x= 1
|1-3|<7
2<7 (sufficient)

(Not sufficient)

Answer is E

Posted from my mobile device


Statement 1 and 2:
0<x<10
Case 1: x=9
|9-3|=7
7=7 (insufficient)

|9-3| is 6 - it's sufficient :) :)

Yes got my mistake
Thank you.
Retired Moderator
Joined: 05 May 2016
Posts: 792
Own Kudos [?]: 683 [0]
Given Kudos: 1316
Location: India
Send PM
Re: Is |x - 3| < 7 ? (1) x > 0 (2) x < 10 [#permalink]
Bunuel wrote:
Is |x - 3| < 7 ?

(1) x > 0
(2) x < 10

DS21224


Is |x - 3| < 7

We get 2 cases.

Case 1: if x - 3 > 0
=> x- 3 < 7
=> x < 10.

Case 2: if x - 3 < 0
=> - x + 3 < 7
=> - x < 4
=> x > -4.

Thus we get : -4 < x < 10.
Now,

Statement 1: x > 0.
It satisfies x< 10.
but what if x > = 10. The the inequality wont hold true.

So Insufficient.


Statement 2: x < 10
It satisfies both : x < 10 x > -4.
But what if x < -4 also,

So this statement is also Insufficient.


Statement 1 + Statement 2:
0 < x < 10.
It satisfies both the sides of Inequality.
Thus Sufficient.


Answer C.
IESE School Moderator
Joined: 11 Feb 2019
Posts: 271
Own Kudos [?]: 171 [0]
Given Kudos: 53
Send PM
Is |x - 3| < 7 ? (1) x > 0 (2) x < 10 [#permalink]
Given: |x - 3| < 7
Lets analyze the qtn statement: x is 7 points away from 3 on the number line
=> -4<x<10

Let analyze the statemnets:
I: x > 0 i.e x can be 3,5,10 or even 20 but we know x<10 NOT SUFFICIENT
II: x < 10 i.e. x can be 5,3,0 ,-4 or even -10 but we know -4<x NOT SUFFICIENT

Lets combine both:

from I: x>0
from II: x<10
==> 0<x<10

This is well within the range from out initial analysis of -4<x<10. Hence SUFFICIENT

C
Intern
Intern
Joined: 17 May 2020
Posts: 2
Own Kudos [?]: 6 [1]
Given Kudos: 43
Location: Viet Nam
Concentration: General Management, Economics
GPA: 4
WE:General Management (Energy and Utilities)
Send PM
Re: Is |x - 3| < 7 ? (1) x > 0 (2) x < 10 [#permalink]
1
Bookmarks
|x - 3| < 7 can be re-written as below:

~ -7<x-3 < 7
~ -4 < x < 10

(1): x>0: x>10 does not make the inequality correct >> INSUFFICIENT
(2): x<10: if x<-4 >> does not make the inequality correct >> INSUFFICIENT
(1) + (2): 0<x<10 (within the range from -4 to 10) is totally Sufficient.

Hence C
VP
VP
Joined: 11 Aug 2020
Posts: 1262
Own Kudos [?]: 201 [0]
Given Kudos: 332
Send PM
Re: Is |x - 3| < 7 ? (1) x > 0 (2) x < 10 [#permalink]
Is |x - 3| < 7 ?

Rephrased: -4 < x < 10?

(1) x > 0
Clearly insufficient.
e.g. x = 11 or 100 then the answer is NO.
e.g. x = 5 then YES

(2) x < 10
Clearly insufficient.
x = 5 then YES
x = -100 then NO

C: Sufficient b/c 0 < x < 10 overlaps with the defined region.
User avatar
Non-Human User
Joined: 09 Sep 2013
Posts: 32653
Own Kudos [?]: 821 [0]
Given Kudos: 0
Send PM
Re: Is |x - 3| < 7 ? (1) x > 0 (2) x < 10 [#permalink]
Hello from the GMAT Club BumpBot!

Thanks to another GMAT Club member, I have just discovered this valuable topic, yet it had no discussion for over a year. I am now bumping it up - doing my job. I think you may find it valuable (esp those replies with Kudos).

Want to see all other topics I dig out? Follow me (click follow button on profile). You will receive a summary of all topics I bump in your profile area as well as via email.
GMAT Club Bot
Re: Is |x - 3| < 7 ? (1) x > 0 (2) x < 10 [#permalink]
Moderator:
Math Expert
92902 posts

Powered by phpBB © phpBB Group | Emoji artwork provided by EmojiOne